Endogenous Retroviruses II

Discussion about scientific issues as they relate to God and Christianity including archaeology, origins of life, the universe, intelligent design, evolution, etc.
NewCreature
Familiar Member
Posts: 32
Joined: Fri Mar 30, 2007 6:32 am

Post by NewCreature »

BGoodForGoodSake wrote:
NewCreature wrote:I don't think it would be wise to start with my assumptions to show my conclusions that would be quite circular. What shall I say I assume this to be true so I will conlude this. Sort of silly. I agree lets just stick to the facts.

While it was once thought that insertion points were quite random viruses show insertion site preference. A lot has been studied about this in particular with HIV.
Let us expand this fact. A retroviral insertion prefers certain base sequences. But of course a virus is not restricted to this, only more likely to insert here. The way you presented it in your posts you made it seem that the preference leads to few possible insertion sites.
It is not a completely unknown that a virus can have such an affinity for a particular insertion point that it will only insert at one location. This can be found in a review of the literature on insertion point affinity in the human genome.

BGoodForGoodSake wrote:
NewCreature wrote:This fact could easily account for viable hosts showing the same viral fragments in the preferred insertion sites.
I asked for scientific assumptions because of statements such as this. It would appear you assumed that preffered sites would limit the number of possible insertion points to the point where this sort of statement can be made.
IT appears that you misunderstand than. Viruses demonstrate that they have an affinity to insert that makes them not random insertions. The statement doesn't have any problems with it. IT is fact that viruses don't insert at completely random locations, and it is even demonstrated from publicly available literature ont he topic that a virus can indeed insert in a SPECIFIC location. I was mearly pointing out the possibility that given that it is not an entirely random process that we would expect to find some data in compared genomes indicating patterns. I think you are simply attacking the position because it doesn't aline with your conclusions. A guided process of insertions of viral matter and a known instance of a specific insertion together with man's ability to make specific and reliable insertions point to the fact that viral matter can insert in very specific ways. The fact that we see some specific insertions in genomes is explained by this hypothesis. I think we could devise other hypothesis that would also explain this.
BGoodForGoodSake wrote:This is simply not the case. The chances of two disparate insertion events occurring at the same site AND THEN finding it's way into every individual in two separate species is highly unlikely.
Yes I agree completley that is why a vast majority of viral insertions do not happen in this way. Way over 99% of viral insertions are more unique than the common ERVs that do show this. IF their is a pandemic where the vast majority of individuals are infected by a small number of viruses that do have high insertion affinities or perhaps even unique ones it is quite possible that that could account for the observations.
BGoodForGoodSake wrote:Again preference is based on structure at the integration sites, it could be anywhere along the genome.
Partly so yes. Numerous repeats may be unsuccessful attempts by the virus to integrate. Viruses are capable of exact insertions, and man is able to do exact insertions with viral matter. Again way over 99% of viruses seem to be able to integrate at numerous spots. In fact the human genome has multiple copies of particular HERVs wouldn't it be likely that these insertions are in decending order of most suitable insertion points?

BGoodForGoodSake wrote:
NewCreature wrote:Some scientists are coming to the conclusion that numerous repeats of viral fragments is actually the virus searching for the best insertion point.
Come again? Please point me to the source. I am not sure this statement retained the original content.
Sorry That isn't a direct quote that is a synthesis of my readings. IT may be more proper to state it as viruses are capable of inserting at various locations. Some of the insertions don't result in a viral infection, but the virus has the ability to insert itself in anough different places that it will end up with a successful viral infection. Although not all insertions are successful, it appears that the virus is able to "find" particular locations. I guess my statement was poor because it endues viruses with the ability to think.
BGoodForGoodSake wrote:Because the insertion is chemically based it is assumed that there could be a preference. But again preference is by structure of the host genome.
Not once has a virus been shown to insert into the same spot in the genome.
Actually this is a false statement. IT is quite rare, but it does happen. Actually is you take some of the higher estimates, some say that the human genome is up to 43% of viral origin. Again the appearence of the distribution of ERVs does show that there are very very few viral insertions that appear in specific locations. Way way less than 1% especially using the higher end of estimates of viral matter in the human genome. Man can do it with relative ease using the viral structure, and it also does accour naturally.

Just few more comments that relate to the rest of your post. I am not ignoring the distribution of ERVs. Every virus has a particular cross section of the species that can host it. I don't see it as a big issue. Sure it is interesteing that there are some viruses that chimps and humans share, I'm just glad we don't share more of them. You are making an argument from air. What does it prove or suggest that more similar species share more ERVs? Species that have more in common with one another being more similar is not unique to any particular theory. What are you suggesting that by your theory ralated organisms being similar is somehow proof of something? Can I use the observation of similarity among more similar animals for my hypothesizing and theorizing too, or must I choose a different set of observations?
BGoodForGoodSake wrote:
NewCreature wrote:IT is also likely that since these bits of code do have use to the organisms that these shared bits could be designed.
This is not a fact. Based on what can you make this conclusion?
How have you disproven that designed organisms wouldn't have these useful bits of information in them? ARE you even being real here? [shakes head in wonder]. What a priori reason do you have to declare that the human genome couldn't have started with some of this DNA? IT appears that you have conlcluded the matter for yourself. I am at a complete loss as to how to even respond to this.
BGoodForGoodSake wrote:
NewCreature wrote:The breeding population should have bred out the individual not the individual breeding out the entire gene pool.
Another assumption. The experiments, mathematics and theories do not support this assumption. Actually I don't know where this idea came from.

This is not an assumption, and you are completley wrong in claiming that mathematics, experiments, and reasonable theorizing lead to the conclusion that one individual will change the entire gene pool. Put 14 mutts in a cage with a purebred labrador and the purebred will dissappear across generations.
BGoodForGoodSake wrote:Let us assume you are correct.
Any different trait will be removed from the gene pool.
Over time a population will consist of genetic clones.
If this were true then every individual would be a clone of the next identical in every way. A quick look at the natural world, and this is falsified immediately.
That's a bit of a stretch, but sexual reproduction and the resistence of dna to change do more so indicate this extreme than they indicate speciation. You are more so arguing for genetic clones than I am. Are you even engaging the subject at all? You propose that all species are genetic clones of an individual in that they all have some sequence in common by inheritance. I think we are getting abit off track. The last few points have simply been attacked rather than investigated. What single new trait from any individual in recorded history has spread throughout the entire genome?

Your simply excusing things as not fact really has no weight whatsoever.
User avatar
BGoodForGoodSake
Ultimate Member
Posts: 2127
Joined: Mon Aug 29, 2005 9:44 am
Christian: No
Location: Washington D.C.

Post by BGoodForGoodSake »

NewCreature wrote:It is not a completely unknown that a virus can have such an affinity for a particular insertion point that it will only insert at one location. This can be found in a review of the literature on insertion point affinity in the human genome.
Could you provide me with a reference?
NewCreature wrote:
BGoodForGoodSake wrote:
NewCreature wrote:This fact could easily account for viable hosts showing the same viral fragments in the preferred insertion sites.
I asked for scientific assumptions because of statements such as this. It would appear you assumed that preffered sites would limit the number of possible insertion points to the point where this sort of statement can be made.
IT appears that you misunderstand than. Viruses demonstrate that they have an affinity to insert that makes them not random insertions. The statement doesn't have any problems with it. IT is fact that viruses don't insert at completely random locations, and it is even demonstrated from publicly available literature ont he topic that a virus can indeed insert in a SPECIFIC location. I was mearly pointing out the possibility that given that it is not an entirely random process that we would expect to find some data in compared genomes indicating patterns. I think you are simply attacking the position because it doesn't aline with your conclusions. A guided process of insertions of viral matter and a known instance of a specific insertion together with man's ability to make specific and reliable insertions point to the fact that viral matter can insert in very specific ways. The fact that we see some specific insertions in genomes is explained by this hypothesis. I think we could devise other hypothesis that would also explain this.
Many ERV's are deactivated, what are the chances that disparate insertions would contain the same defects? Also keep in mind that ERV stands for endogenous retroviruses. A retrovirus is a special class of virus. Don't confuse this with virus' in general.
NewCreature wrote:Yes I agree completley that is why a vast majority of viral insertions do not happen in this way. Way over 99% of viral insertions are more unique than the common ERVs that do show this. IF their is a pandemic where the vast majority of individuals are infected by a small number of viruses that do have high insertion affinities or perhaps even unique ones it is quite possible that that could account for the observations.
How do multiple insertion events explain the shared ERV's? This seems a bit of an over simplification.
NewCreature wrote:Partly so yes. Numerous repeats may be unsuccessful attempts by the virus to integrate. Viruses are capable of exact insertions, and man is able to do exact insertions with viral matter.
Are we talking about viruses in general or retroviruses?
NewCreature wrote:Again way over 99% of viruses seem to be able to integrate at numerous spots. In fact the human genome has multiple copies of particular HERVs wouldn't it be likely that these insertions are in decending order of most suitable insertion points?
Possibly. Is it possible to supply a source for the 99% figure? Additionally what is the purpose of bringing up this possibility?
NewCreature wrote:Sorry That isn't a direct quote that is a synthesis of my readings. IT may be more proper to state it as viruses are capable of inserting at various locations. Some of the insertions don't result in a viral infection, but the virus has the ability to insert itself in anough different places that it will end up with a successful viral infection. Although not all insertions are successful, it appears that the virus is able to "find" particular locations. I guess my statement was poor because it endues viruses with the ability to think.
BGoodForGoodSake wrote:Because the insertion is chemically based it is assumed that there could be a preference. But again preference is by structure of the host genome.
Not once has a virus been shown to insert into the same spot in the genome.
Actually this is a false statement. IT is quite rare, but it does happen. Actually is you take some of the higher estimates, some say that the human genome is up to 43% of viral origin. Again the appearence of the distribution of ERVs does show that there are very very few viral insertions that appear in specific locations. Way way less than 1% especially using the higher end of estimates of viral matter in the human genome. Man can do it with relative ease using the viral structure, and it also does accour naturally.
It seems here that you are stating that viral insertions are now non-targeted?
NewCreature wrote:Just few more comments that relate to the rest of your post. I am not ignoring the distribution of ERVs. Every virus has a particular cross section of the species that can host it. I don't see it as a big issue. Sure it is interesteing that there are some viruses that chimps and humans share, I'm just glad we don't share more of them.
We are not talking about actual live viruses, we are talking about ERV's.
NewCreature wrote:You are making an argument from air. What does it prove or suggest that more similar species share more ERVs? Species that have more in common with one another being more similar is not unique to any particular theory. What are you suggesting that by your theory ralated organisms being similar is somehow proof of something? Can I use the observation of similarity among more similar animals for my hypothesizing and theorizing too, or must I choose a different set of observations?
You can use the same set of observations. But the patterns must be accounted for. Why do all these species share an insertion pont, and why do the sequences display the pattern of differences found?
NewCreature wrote:How have you disproven that designed organisms wouldn't have these useful bits of information in them?
You must first prove they were designed. The idea that they are designed is an assumption no?
NewCreature wrote:ARE you even being real here? [shakes head in wonder]. What a priori reason do you have to declare that the human genome couldn't have started with some of this DNA?
The reason is the similarity of these ERV's to retroviral genetic code. You yourself used the figure that the human genome consists of 43% viral material. These two assertions are a bit contradictory, don't you concur?
NewCreature wrote:IT appears that you have conlcluded the matter for yourself. I am at a complete loss as to how to even respond to this.
Simple, you stated that the the reason that there are similarities is because the bits of code are useful and thus the DNA was designed this way.
You listed this among several facts.
However this could only be a fact if it was shown that DNA was designed, and that all these bits of code are useful. Is this the case?
Therefore I stated that this was not in fact a fact.
NewCreature wrote:This is not an assumption, and you are completley wrong in claiming that mathematics, experiments, and reasonable theorizing lead to the conclusion that one individual will change the entire gene pool. Put 14 mutts in a cage with a purebred labrador and the purebred will dissappear across generations.
Ah, but the genes which carried the traits of the purebred are still within the populations gene pool. They are not removed. Only distributed among the mutts within the population.

Also you forget that the mutts are under different selective pressures.

I can assure you that if we then take this mutt population and bring them back into the selective pressures which created the purebred, that after enough generations, we will have a population similar to the pure bred labrador. This is because the traits are still within the gene pool.
NewCreature wrote:That's a bit of a stretch, but sexual reproduction and the resistence of dna to change do more so indicate this extreme than they indicate speciation. You are more so arguing for genetic clones than I am. Are you even engaging the subject at all?
Yes I am,
you stated "The breeding population should have bred out the individual not the individual breeding out the entire gene pool."

This ignores that a trait can indeed spread throughout a population if it confers selective advantage.

It also show a misunderstanding of genetics. Individuals are not bred out, the new trait becomes a part of the populations gene pool.

I simply took your argument and built on it. If it were truly the case that individual traits were bred out, then eventually you will be reduce to a homogenous population. Do you see a problem with this conclusion? Eventually each individual would be indistinguishable from the next.
Is there a problem with this reasoning?
So therefore I tried to demonstrate in laymen's terms that the entire concept of breeding out is not a reflection of reality.
This concept may come from breeders who are unfamiliar with the genetic underpinnings of the selective breeding process.
NewCreature wrote:You propose that all species are genetic clones of an individual in that they all have some sequence in common by inheritance. I think we are getting abit off track. The last few points have simply been attacked rather than investigated. What single new trait from any individual in recorded history has spread throughout the entire genome?
In breeding programs where do the new traits come from? And how does this trait spread throughout the entire purebred population?
NewCreature wrote:Your simply excusing things as not fact really has no weight whatsoever.
Are you saying that differential selection does not exist?

Let's hypothesize for a moment.
Then we can get back to the observations.

Lets say that we have a population of kangaroos which have been released into a new environment. Each kangaroo is unique in it's own way. Does each kangaroo have the exact same likelyhood of reaching maturation?

If they do not what will be the result after many, many generations in which a slight advantage would be amplified?

This might be going off topic. If you choose to continue this particular line of reasoning I will respond to this section on a different thread.
It is not length of life, but depth of life. -- Ralph Waldo Emerson
NewCreature
Familiar Member
Posts: 32
Joined: Fri Mar 30, 2007 6:32 am

Post by NewCreature »

A few comments.

Sorry I can't find the article I was reading on the virus with the unique insertion point. IT may not have been a retrovirus; I'm not sure. Which of course doesn't mean that ERVs can't wind up in the same location. The shared ERVs could be considered as evidence that viruses are able to integrate at specific sites. It is clear that Retroviruses do display insertion point affinity, this coupled with the site specific integrases and recombinases, helping to guide the process, these things could easily account for some retroviruses appearing in the same location. This is especially true in the light of the large number of instances of these ERVs in similar species. Given the affinity for appropriate locations and the large number of infections the likelihood of a retrovirus finding the most suitable locations for integration goes up.

Scientists have been able to re-synthesize HERV-K and found that it will reintegrate with the same sequence that now appears in the human genome. With about 30,000 different ERV infections in both chimps and humans it is highly likely that the most favorable site for a particular virus will become infected. If you take the number of shared ERVs and divide them by the total number of ERV infections in the genome you will find that 99% of them are not shared. While retroviruses do have an affinity to integrate at certain locations, not all integrations will result in a viral outbreak. It would seem that the viruses have the ability to integrate at different locations in a guided manner so that results in them being inserted in specific location eventually.

Numerous infections among viable hosts during a pandemic explain the matter as well as the common ancestor. It wouldn't be possible for me to prove this to be true any more than it would be possible for you to prove the common ancestor explanation. IT is just the case that in years gone by it was the assumption that retroviral infections were random, and that all the HERVs were the result of infections. Today we find that it is a guided process, and that these proteins are important to the body. Seems like those early predictions have failed.
User avatar
BGoodForGoodSake
Ultimate Member
Posts: 2127
Joined: Mon Aug 29, 2005 9:44 am
Christian: No
Location: Washington D.C.

Post by BGoodForGoodSake »

NewCreature wrote:A few comments.

Sorry I can't find the article I was reading on the virus with the unique insertion point. IT may not have been a retrovirus; I'm not sure. Which of course doesn't mean that ERVs can't wind up in the same location. The shared ERVs could be considered as evidence that viruses are able to integrate at specific sites. It is clear that Retroviruses do display insertion point affinity, this coupled with the site specific integrases and recombinases, helping to guide the process, these things could easily account for some retroviruses appearing in the same location.
I will assume from the tone of this post that you concede the retroviral origins of ERV's(1)

First lets not overstate insertion point affinity to mean a very few possible insertion sites.
Second lets consider that these retroviruses are in every individual in both species. When did the insertion occur?
NewCreature wrote:This is especially true in the light of the large number of instances of these ERVs in similar species. Given the affinity for appropriate locations and the large number of infections the likelihood of a retrovirus finding the most suitable locations for integration goes up.
Insertion points may match up but genetic sequences are highly unlikely to match up as closely as we observe. Nor should we expect to find that the differences would fall into a pattern.(2)
NewCreature wrote:Scientists have been able to re-synthesize HERV-K and found that it will reintegrate with the same sequence that now appears in the human genome.
Please cite this article, and please let me know if they stated that this resynthesized viral material will integrate at this site exclusively or at least has a statistically high chance of integrating at this site.
NewCreature wrote:With about 30,000 different ERV infections in both chimps and humans it is highly likely that the most favorable site for a particular virus will become infected. If you take the number of shared ERVs and divide them by the total number of ERV infections in the genome you will find that 99% of them are not shared.
Where does this figure come from?

Having said that you will find that the one's that are shared between the various species fall into a discernable pattern. There are no shared ERV's between Humans and Gorillas which are not shared by Chimps as well.

While on the other had you do find instances of non shared(homologous) ERV's which are shared between Chimpanzees and Gorillas but not in Humans. In this case due to the differences in the sequence and insertion site we are able to determine that these were the result of separate insertion events.(3)

If it were the case that ERV's which are shared is due to separate infection events and fall into a pattern because similar species are more likely to be infected by the same viral vector than how can the above be explained?

What is the probability that such random insertions from multiple insertion events fall into such a pattern?
NewCreature wrote:While retroviruses do have an affinity to integrate at certain locations, not all integrations will result in a viral outbreak. It would seem that the viruses have the ability to integrate at different locations in a guided manner so that results in them being inserted in specific location eventually.
You seem to be addressing insertion points only without focuising on the sequences of these insertions.

However let us examine a specific instance of an ERV found in Old World Monkeys and African Great Apes, but not in Humans, and Orangutans.
This ERV specified as PTERV1 may seem to undermine the conclusion based on statistical analysis of previous studies. Namely that ERV similarities between species point to a common ancestor.

"[researchers] found over 100 copies of PTERV1 in each African ape (chimp and gorilla) and Old World monkey (baboon and macaque) species. The authors compared the sites of viral integration in each of these primates and found that few if any of these insertion sites were shared among the primates."(3)

In addition the genetic sequence is not conserved between the Old world Monkeys and Great Apes.
Finally these infections do not show the pattern discussed in earlier posts.

Why if the insertion of viral material is bound to share insertion points among various species does the pattern not match for separately integrated material.

For instance should we not expect SIV and HIV to share identical insertion points eventually? And should we expect their sequences to be similar.

Yet we find that SIV and HIV have a multitude of insertion sites. And they do not have similar enough sequences to juistify the conclusion that ERV resulting from SIV and one resulting from HIV is a result of common ancestry. In fact a scientist would conclude them to be of separate infection events.
NewCreature wrote:Numerous infections among viable hosts during a pandemic explain the matter as well as the common ancestor. It wouldn't be possible for me to prove this to be true any more than it would be possible for you to prove the common ancestor explanation.
We can disprove it easily by showing that the sequences of the shared ERV's show much too much similarity to be attributed to separate insertion events. They are noted as homologous because their genetic sequence matches up, not only due to the shared insertion sites.
NewCreature wrote:IT is just the case that in years gone by it was the assumption that retroviral infections were random, and that all the HERVs were the result of infections. Today we find that it is a guided process, and that these proteins are important to the body. Seems like those early predictions have failed.
This is not the case, it is still the case that a retrovirus has many possible insertion sites. And the site in which it inserts is statistacilly random. But the focus need to be on the sequences themselves and the pattern that an analysis of these sequences reveal.(4)

1. Boeke, J. D. & Stoye, J. P. (1997) in Retroviruses, eds. Coffin, J. M., Hughes, S. H. & Varmus, H. E. (Cold Spring Harbor Lab. Press, Plainview, NY), pp. 343-436.
2. Coffin, J. M. (1996) in Fundamental Virology, eds. Fields, B. N., Knipe, D. M. & Howley, P. M. (Lippincott-Raven, Philadelphia), pp. 763-844.
3. PLoS 2005 March 1. doi: 10.1371/journal.pbio.0030126.
4. Brown, P. O. (1997) in Retroviruses, eds. Coffin, J. M., Hughes, S. H. & Varmus, H. E. (Cold Spring Harbor Laboratory Press, Plainview, NY).
NewCreature
Familiar Member
Posts: 32
Joined: Fri Mar 30, 2007 6:32 am

Post by NewCreature »

BGoodForGoodSake wrote: I will assume from the tone of this post that you concede the retroviral origins of ERV's. First lets not overstate insertion point affinity to mean a very few possible insertion sites. Second lets consider that these retroviruses are in every individual in both species. When did the insertion occur?
No I do not concede the retroviral origin of ERVs. IT is often stated that 43% of the human genome are retro-elements. There is no a priori reason to conclude that the human genome doesn't have some of these sequences naturally occurring outside of viral infections. I think that it is clear that some of the ERVs are from viral infection of gametes, but that doesn't lead to the conclusion that they all are. I'm not overstating insertion point affinity, I am simply pointing out that what was once thought to be random is being shown more and more to be a guided process. There are site specific tranposases, recombinases, and integrases. These factors are at work during the propagation of a viral infection throughout the host cell. When the virus retrotransposes in numerous places throughout the host gamete it has been suggested that this is a viral mechanisms that is “searching” for specific integration sites.

IT seems that the most likely explanation would be to theorize that infection occurred during a pandemic when many individuals capable of hosting the virus were infected. Again with a directed process of integration we would expect to find patterns. We would not expect to find infections in species that cannot host the virus. Your argument is: “well it is not a random distribution so multiple infections don't work”, but it is being shown more and more everyday that it is a guided process and this determines we will find patterns.
BGoodForGoodSake wrote: Insertion points may match up but genetic sequences are highly unlikely to match up as closely as we observe. Nor should we expect to find that the differences would fall into a pattern.
Why?
BGoodForGoodSake wrote:
NewCreature wrote:Scientists have been able to re-synthesize HERV-K and found that it will reintegrate with the same sequence that now appears in the human genome.
Please cite this article, and please let me know if they stated that this resynthesized viral material will integrate at this site exclusively or at least has a statistically high chance of integrating at this site.
I don't think the delibertley infected anyone to take advantage of seeing all the viral mechanisms at work. Here is an abstract, I can't find the exact article I was reading. http://www.genome.org/cgi/content/abstract/16/12/1548
from the abstract wrote:This element, Phoenix, produces viral particles that disclose all of the structural and functional properties of a bona-fide retrovirus, can infect mammalian, including human, cells, and integrate with the exact signature of the presently found endogenous HERV-K progeny.
Same virus; same sequence

BGoodForGoodSake wrote:
NewCreature wrote:With about 30,000 different ERV infections in both chimps and humans it is highly likely that the most favorable site for a particular virus will become infected. If you take the number of shared ERVs and divide them by the total number of ERV infections in the genome you will find that 99% of them are not shared.
Where does this figure come from?
Try it for yourself.
BGoodForGoodSake wrote: Having said that you will find that the one's that are shared between the various species fall into a discernable pattern. There are no shared ERV's between Humans and Gorillas which are not shared by Chimps as well.
So What's your point? IF a virus is capable of infecting a human and a dissimilar species like a gorilla it will be very likely of infecting the more similar chimp. Yes the discernable pattern is a result of the guided process; it is not random and we expect a pattern.
BGoodForGoodSake wrote: While on the other hand you do find instances of non shared(homologous) ERV's which are shared between Chimpanzees and Gorillas but not in Humans. In this case due to the differences in the sequence and insertion site we are able to determine that these were the result of separate insertion events. .
How are you able to determine that? I am not saying that it is not so, but how can you show that the same event didn't retrotranspose with a unique sequence and into specific locations within the genome of those two whereas mechanisms differ slightly in the human. Also perhaps the human was infected but that virus never made it into a gamete that became an individual. I think you are right that it was an infection by a virus that isn't capable of infecting the human genome. But to just state it as fact without taking into account the site specific tranposases, integrases, and recombinases within the genome of the chimp and gorilla and how those differ from the human, why you are just hypothesizing.

BGoodForGoodSake wrote: If it were the case that ERV's which are shared is due to separate infection events and fall into a pattern because similar species are more likely to be infected by the same viral vector than how can the above be explained?
WE could offer several potential explanations. Here are two. Separate events where in one virus the gorilla was immune and in the other the human was immune. Same event but unique integrases, recombinants, etc resulted in disparate viral signatures. Perhaps if you would like to take the time we can come up with several other possibilities. You seem rather to hold to a position rather than to really engage and inquire.
BGoodForGoodSake wrote: What is the probability that such random insertions from multiple insertion events fall into such a pattern?
What is the probability that an apple is an orange? It is not a random process. The virus has an affinity for similar locations, and mechanisms exist to guide the process as the virus retrotransposes throughout the host cell.
BGoodForGoodSake wrote:
NewCreature wrote:While retroviruses do have an affinity to integrate at certain locations, not all integrations will result in a viral outbreak. It would seem that the viruses have the ability to integrate at different locations in a guided manner so that results in them being inserted in specific location eventually.
You seem to be addressing insertion points only without focuising on the sequences of these insertions.
Yes that is right. One thing at a time (sort of). Please see the reconstructed retrovirus “phoenix”.

To try and sum up I will make some comments in response without further quotes. IT is quite likely that sequences wouldn't be the same for different viruses. Each virus will integrate in a unique way. This is why the patterns do not match for separately integrated material. Further a review of genomes will show you that most of the time viruses appear to be separate and isolated incidences. This is why a pandemic is the likely cause of the same sequences in the same location.

You say you can handily disprove this. Perhaps we will just have the jury disregard that statement if you intend to leave it empty. Same virus leads to same sequence. Same mechanisms with numerous sites leads to a high probability of a pattern developing resulting in instances of shared sites.

This guided process is not statistically random, although I know that from the early days it has been the prediction of evolutionists that it would be found to be so. We find that individuals and species do in fact share sequences and share insertion points. Viruses have an affinity for an insertion point and there are site specific recombinases, integrases, etc. This is a guided process and insures that specific locations will eventually be found by the viral outbreak.
User avatar
BGoodForGoodSake
Ultimate Member
Posts: 2127
Joined: Mon Aug 29, 2005 9:44 am
Christian: No
Location: Washington D.C.

Post by BGoodForGoodSake »

NewCreature wrote:I'm not overstating insertion point affinity, I am simply pointing out that what was once thought to be random is being shown more and more to be a guided process. There are site specific tranposases, recombinases, and integrases. These factors are at work during the propagation of a viral infection throughout the host cell. When the virus retrotransposes in numerous places throughout the host gamete it has been suggested that this is a viral mechanisms that is “searching” for specific integration sites.
This is a good description.
NewCreature wrote:IT seems that the most likely explanation would be to theorize that infection occurred during a pandemic when many individuals capable of hosting the virus were infected. Again with a directed process of integration we would expect to find patterns. We would not expect to find infections in species that cannot host the virus. Your argument is: “well it is not a random distribution so multiple infections don't work”, but it is being shown more and more everyday that it is a guided process and this determines we will find patterns.
But the problem here is that the insertion is found in every individual of a species with nearly identical sequences. Not withstanding the identical insertion points between species.
NewCreature wrote:
BGoodForGoodSake wrote: Insertion points may match up but genetic sequences are highly unlikely to match up as closely as we observe. Nor should we expect to find that the differences would fall into a pattern.
Why?
Here is the reasoning why.
Human genetic material contains ERV's.
There is a set of ERV's shared by all Human Beings.
Your assumption is that some of these were part of the human genome from the beginning by design, and others were aquired via retroviral vectors.
How do we differentiate those which were not there from the beginning and those which are? And shouldn't all have been there from the start because they are found in all human beings?
The same can be said for Chimpanzees and Gorillas.
Yet don't these elements consist of genes which encode for retroviral proteins?
Why discard the possibility that ERV's must be the result of viral insertion.
Can we truly dismiss this by saying that there must be some purpose beneficial to the host, simply because we want to assume design? Can you dismiss a possibility in this manor? Or should we at least entertain the posibility that these are the result of viral infections, based on observations of retroviral action which leads to identical results in somatic cells?
We must admit that somatic injection of retroviral material leads to similar results, and in some cases ERV's still produce retroviral particles, as you have linked to in the previous post.
So now let us switch our views and submit that these shared ERV's are the result of disparate viral injection events. There are several ERV's shared between the three species. In each case the sequences are close. So close in fact that if the only explanation left besides common descent is an intraspecies pandemic.
But lets note the observed patterns. Chimpanzee insertions are more similar in sequence to Humans than that of Gorilla insertions, in every case. How can this be if the insertions occurred during an intra species pandemic?
Would it not sometimes be the case where Gorillas and Chimpanzee sequences are more similar than Humans, or Humans and Gorillas more similar than that of Chimpanzees? Out of three possibilities all cases fall into the observed case, how can this be?
Remember we are talking about a single intra-species pandemic.
To repeat myself I understand how similar species would be subjected to infections from a single virus or a family of viruses, but we should expect an even distribution of sequence variance, not the patterned one we observe.

And finally how did this insertion make it's way into every modern Human, Gorilla, and Chimpanzee? And maintain their sequence similarities.

Remember retroviral genetic sequences tend to mutate at high rates. Yet the insertions found in every individual within a species, is nearly identical.
And similar across species.

Now you have been using three options that are distinct and come with their own problems.
posibility A: ERV's are not the result of retroviral insertions but are designed into the genome.
posibility B: ERV's are the result of multiple disparate insertion events.
posibility C: ERV's are the result of targeted ERV insertions so identical insertion point are bound to happen.

You cannot combine elements from the three possibilities to combat the questions posed in previous threads. As they are not the same solutions.

For instance when faced with identical insertion points, posibility B was given as a solution, that given enough insertions some are bound to match up.
However when data shows that the sequences are similar, you use posibility C to point out that the material is injected in a targeted fashion.
And when the fact that sequence variance shows a pattern posibility A is brought up.

These three solutions cannot all coexist unless there is something fundamental I have missed in your posts.
One of these solutions you are proposing should be able to fit the observations.
NewCreature wrote:
BGoodForGoodSake wrote:
NewCreature wrote:Scientists have been able to re-synthesize HERV-K and found that it will reintegrate with the same sequence that now appears in the human genome.
Please cite this article, and please let me know if they stated that this resynthesized viral material will integrate at this site exclusively or at least has a statistically high chance of integrating at this site.
I don't think the delibertley infected anyone to take advantage of seeing all the viral mechanisms at work. Here is an abstract, I can't find the exact article I was reading.
There's no need to infect individuals, they can cause the viral particles to infect tissue samples in a culture.
NewCreature wrote:http://www.genome.org/cgi/content/abstract/16/12/1548
from the abstract wrote:This element, Phoenix, produces viral particles that disclose all of the structural and functional properties of a bona-fide retrovirus, can infect mammalian, including human, cells, and integrate with the exact signature of the presently found endogenous HERV-K progeny.
Same virus; same sequence
But no mention of same insertion point. What this shows is that HERV-K is very likely the result of a viral insertion.
Here you are describing posibility B, that eventually given enough time disparate events among the various species will lead to cases where material is injected in identical insertion points. However in this experiment the viral particles were reconstructed from the ERV itself.
The reconstruction was validated as an acurate reconstruction when it was allowed to reintegrate into the genome and left an identical signature.

However in the wild this viral particle is free to collect mutations during replication, the signature will change with each generation of viral particles.
The infections caused in disparate events will also have dissimilar signatures.
The distribution of cross species analysis should display balanced distribution.
Meaning given three species A, B and C that in some cases insertion sequence is more similar between species A and B than C, more similar between B and C than A, and in other cases more similar between A and C rather than B.
However the data shows that the sequences between species A and B is always more similar than that of species C.
NewCreature wrote:
BGoodForGoodSake wrote: Where does this figure come from?
Try it for yourself.
So here you are again pointing to posibility B. That the vast majority of ERV's are not shared it is bound to happen that some will be similar. Again this would explain the similarities in insertion points but not the similarity in sequence.
NewCreature wrote:
BGoodForGoodSake wrote: Having said that you will find that the one's that are shared between the various species fall into a discernable pattern. There are no shared ERV's between Humans and Gorillas which are not shared by Chimps as well.
So What's your point? IF a virus is capable of infecting a human and a dissimilar species like a gorilla it will be very likely of infecting the more similar chimp. Yes the discernable pattern is a result of the guided process; it is not random and we expect a pattern.
Explain how this would occur.
Given the mutational rates of viruses.
(The sequence of retroviral RNA is always mutating)
Why would it never occur that a given sequence is more similar between Gorillas and Humans than Chimpanzees and Humans? Does the virus always infect the Gorilla last?
NewCreature wrote:
BGoodForGoodSake wrote: While on the other hand you do find instances of non shared(homologous) ERV's which are shared between Chimpanzees and Gorillas but not in Humans. In this case due to the differences in the sequence and insertion site we are able to determine that these were the result of separate insertion events. .
How are you able to determine that? I am not saying that it is not so, but how can you show that the same event didn't retrotranspose with a unique sequence and into specific locations within the genome of those two whereas mechanisms differ slightly in the human. Also perhaps the human was infected but that virus never made it into a gamete that became an individual. I think you are right that it was an infection by a virus that isn't capable of infecting the human genome. But to just state it as fact without taking into account the site specific tranposases, integrases, and recombinases within the genome of the chimp and gorilla and how those differ from the human, why you are just hypothesizing.
This can be determined because the integration sites of this insertion are not the same. And the sequence differences are far greater pointing to disparate insertion events. In other words the data aligns well with disparate insertion events given our current knowledge of retroviral activity.
The virus must have infected one species and then the other and mutated within their host populations in isolation. And when the viral material finally made it's way into each species genome, the differences in sequence align well with expected mutation rates of retroviruses.
Exactly what would be predicted by option B from above.
NewCreature wrote:
BGoodForGoodSake wrote: If it were the case that ERV's which are shared is due to separate infection events and fall into a pattern because similar species are more likely to be infected by the same viral vector than how can the above be explained?
WE could offer several potential explanations. Here are two. Separate events where in one virus the gorilla was immune and in the other the human was immune. Same event but unique integrases, recombinants, etc resulted in disparate viral signatures. Perhaps if you would like to take the time we can come up with several other possibilities. You seem rather to hold to a position rather than to really engage and inquire.
Let us consider these posibilities.
In these posibilities you hold that one of the species is immune. Thus resulting in the PTERV case. This is fine.
:arrow: But that does not explain why there are no shared insertion points between the remaining two species.
:arrow: And this does not explain why the insertions in the remaining species have such sequence variances.
:arrow: Nor does this explain why in cases where the three species do share an insertion point the sequences vary by much less than expected by independant insertion events.
:arrow: And neither does this explain why in cases where the three species do share an insertion point the patterns are always the same.
Also lets refer back to the ERV-K Phoenix viral particle. The phoenix insertion exhibited the exact same signatures in hamster, cat and human cells. This would seem to be counter to your assertion that the same viral class would exhibit different insertional signatures because of the uniqueness of each species genome.
NewCreature wrote:
BGoodForGoodSake wrote: What is the probability that such random insertions from multiple insertion events fall into such a pattern?
What is the probability that an apple is an orange? It is not a random process. The virus has an affinity for similar locations, and mechanisms exist to guide the process as the virus retrotransposes throughout the host cell.
That is solution C, but my statement was in responce to the following statement.
"With about 30,000 different ERV infections in both chimps and humans it is highly likely that the most favorable site for a particular virus will become infected. If you take the number of shared ERVs and divide them by the total number of ERV infections in the genome you will find that 99% of them are not shared."
In this case you are saying that there are many likely insertion points, ie possibility B. And that the sheer number of insertions will lead to shared insertion points. You are saying that the matching insertions are a chance occurrence given the large number of unshared insertions. So again I will ask, what are the chances of random insertion events leading to similar sequences in these identical insertion points?

And if we are not putting any significance in the number of infections what is the significance of the 99% figure? This was the figure that this portion of the conversation stems from.
NewCreature wrote:
BGoodForGoodSake wrote:
NewCreature wrote:While retroviruses do have an affinity to integrate at certain locations, not all integrations will result in a viral outbreak. It would seem that the viruses have the ability to integrate at different locations in a guided manner so that results in them being inserted in specific location eventually.
You seem to be addressing insertion points only without focuising on the sequences of these insertions.
Yes that is right. One thing at a time (sort of). Please see the reconstructed retrovirus “phoenix”.
Phoenix addresses sequence, the sequence of a reconstructed virus which has not had the chance to mutate freely in the wild. Therefore sequence was a result of the transcription process only. No mutations were involved and thus this could not characterize disparate insertion events. Additionally in the paper you linked to there were no shared insertion points.
"Finally, we mapped the three Phoenix insertion sites using the UCSC Genome Browser and found that integration occurred within (for two of three) or close to (<20>100 kb on each side), four being in the vicinity (<20 kb) of genes, and only two inserted within genes (M. Dewannieux, unpubl.)."(1).
As you can see here insertion preference is for transcribed regions. And in the relatively small sample none of the insertions occurred in identical locations. So in this case you addressed sequence, however in a misguided way, and did not address insertion location. Remember the key is location AND sequence.
NewCreature wrote:To try and sum up I will make some comments in response without further quotes. IT is quite likely that sequences wouldn't be the same for different viruses. Each virus will integrate in a unique way. This is why the patterns do not match for separately integrated material. Further a review of genomes will show you that most of the time viruses appear to be separate and isolated incidences. This is why a pandemic is the likely cause of the same sequences in the same location.
Sequences are not the same for different viruses by definition. Each class of virus integrates in a unique way, not each individual virus of the same class. In other words all deltaretroviruses for example insert their genetic material in more or less the exact same manor.
Slight variations can arise due to mutations in the ENV encoding protein.
Revisiting the pandemic possibility, you say that this explains similar sequence in the similar location, but have yet to address the distribution of similarities. Why are Chimpanzee sequences always more similar to Human than Gorrillas are to Humans if the infection is the result of a single pandemic episode?
NewCreature wrote:You say you can handily disprove this.
I did?
NewCreature wrote:Perhaps we will just have the jury disregard that statement if you intend to leave it empty. Same virus leads to same sequence. Same mechanisms with numerous sites leads to a high probability of a pattern developing resulting in instances of shared sites.
Yes but what pattern would we expect? I will repeat again
should we not expect an even distribution in which in a third of the cases the Human and Chimpanzee sequences are the most similar, and in another third the Human and Gorilla sequences are the most similar and finally in the last third the Chimpanzee and Gorilla sequence is the most similar? Also this is ignoring all of the other species which also share the same insertion points and have the same pattern of sequence discrepancies.
NewCreature wrote:This guided process is not statistically random, although I know that from the early days it has been the prediction of evolutionists that it would be found to be so.
Site this please, as far as I am aware insertion point affinity is not a new concept.
Don't confuse affinity with limited possibilities. Affinity still leaves multiple possible insertion points, where among the preferred sites the viral material will insert is statistically random.
NewCreature wrote:We find that individuals and species do in fact share sequences and share insertion points. Viruses have an affinity for an insertion point and there are site specific recombinases, integrases, etc. This is a guided process and insures that specific locations will eventually be found by the viral outbreak.
Again you are pointing to possibility B. It seems that the majority of your points refer to this posibility.
Namely that ERV's are the result of multiple disparate insertion events.
Let's then focus in on this hypothesis.
:arrow: How then does the insertion get propagated to every individual in the species?

Isn't it true that as time goes on that viral material will change at a much faster clip than the genetic material found in mammals? Proof for this can be found in the fact that HERV's within Humans are close to identical in all members.
:arrow: If the above is true than how much time has to elapse in order for the species involved to be infected? Keep in mind that HERV-K homologs are found in New World Monkeys, Old World Monkeys, Great Apes and Humans all in the exact same insertion point.

:arrow: How likely is it that each species fix the same insertion within the same time period within each respective genepool?
Why the same timeperiod? because as time goes on the genetic code of the virus become less and less like that of the original virus.

1. Dewannieux, M, Harper, F, Richaud, A, Letzelter, C, Ribet, D, Pierron, G & Heidmann, T. (2006) Genome Res 16, 1548—1556.
NewCreature
Familiar Member
Posts: 32
Joined: Fri Mar 30, 2007 6:32 am

Post by NewCreature »

Thanks for the reply. There is quite a bit to digest, so I will not go point for point with quotations, but will try to address things from the top down in response. If I miss something that was of particular interest to you that you want to see addressed just re-post your key point.

I incorporated a few of your statements into this response

The problem here is that the insertion is found in every individual of a species with nearly identical sequences. Not withstanding the identical insertion points between species. I see this as more problematic for common ancestry hypothesis and perhaps even for the generally accepted label of many of these sequences as ERVs. Common ancestry would say that many of these infections are well over 100,000 generations old. Given the highly error prone process of reverse transcription and the instability of retroviral sequences I would wonder how you account for the remarkable stability of these sequences and how phoenix can show the same sequence today? IT would appear that remarkable few mutations have occoured and that these insertions seem to have happened very recently

I don't assume that some of the retroelements are original in a created being I simply point out the fact that there is no reason to arbitrarily deny that this may be so. I discard the possibility that all ERVs MUST be from viral infections, but would readily concede that they might be, and that many seem clearly to be from viral infections. Some sequences code for proteins and some do not. I am not assuming that there must be some benefit to the individual I am demonstrating from observations that many are beneficial. Many scientists are coming more and more to the conclusions that retroelements play roles in viral immunity.

You argue that I should entertain the possibility that ERVs are the result of viral infections because we see retroviral infections yielding identical results in somatic cells. I readily entertain the possiblity. I would even go as far as saying that I suspect that it is highly likely that many, perhaps even most, of these retroelements are from retroviral infection and from retrotransposistion and other essentially unchanged mechanisms of viral integration into the host cell. I further feel that this is what causes the patterns and distributions we observe.


The human and chimp genomes are more identical than the human and gorilla genomes are. The chimp and gorilla genomes are more identical than the gorilla and human ones are. Any of these so called retro elements that are in these three distinct creatures by their design would follow the same pattern as there larger genome, or at least it seems reasonable to conclude this. Further reason would dictate that the viral behavior and integrations in the genomes would still yet follow that same pattern. Indeed we find this to be true.

I would NOT suggest that up to this old age of the world that there has only ever been one intra or inter species pandemic. IT is likely that numerous widespread retroviral infections have taken place. I'm not sure why a unique virus infecting many individuals offering a louder signal to the gene pool, integrating itself in an orderly way would result in an even distribution of sequence variance, not the patterned one we observe. A guided process operating by specific mechanisms integrating into specific and sometimes similar substrate will indeed yield a patterned distribution. I have repeatedly offered the mechanisms whereby the insertion makes it's way into every modern Human, Gorilla, and Chimpanzee? And maintain their sequence similarities, but I have yet to see it addressed how this came to be from common ancestry. You suggest 1 individual and how many millions of years? I am quite interested in how this might come to be from common ancestry. Your time frame has a greater problem with retroviral genetic sequences that tend to mutate at high rates. How can the insertions be found in every individual within a species, and for it to still be nearly identical?

posibility A: ERV's are not the result of retroviral insertions but are designed into the genome. I never proposed this, I think it is clear that at least some retroelements are from viral origins. I just pointed out the fact that this does not demand the conclusion that all retroelements are viral.

I see no need to split up posibility B: ERV's are the result of multiple disparate insertion events. And posibility C: ERV's are the result of targeted ERV insertions so identical insertion point are bound to happen.



I think it has been demonstrated that the interaction between cellular mechanisms and viral mechanisms are capable of encoding very specific sequences, and yielding specific insertions. Possibility B relates closely to the pandemic and results in a similar sequence, and possibility C relates to the guided process of retroviral infection involving site specific tranposases, recombinases, and integrases propagating the viral infection throughout the host cell retrotransposing the sequence into numerous places yielding specific integration sites. The specific ordered mechisms exist independant of pandemic cause or from an isolated incident in a common ancestor.

These two processes work together to yield the pattern we see. IT is not surprising that viruses act more similar in more similar substrate.

I am interested in this . . .
BGoodForGoodSake wrote: The phoenix insertion exhibited the exact same signatures in hamster, cat and human cells. This would seem to be counter to your assertion that the same viral class would exhibit different insertional signatures because of the uniqueness of each species genome.
I am unfamiliar with this observation; do you have any more information on this?. I would suggest that isolated cells in a petri dish do not behave in the same ways as an organism. I would suggest the matching insertions are not a chance occurrence. I would suggest that the large number of unshared insertions indicates that retroviral infection can take place at various points along the genome, and that through the mechanisms of retroviral infection the retroviral infection will find its way to the best possible site for integration. You continue to ask, what are the chances of random insertion events leading to similar sequences in these identical insertion points? An initial retroviral insertion is a small part of the viral infection process, and many of the mechanisms involved in viral activity within the host cells are still poorly understood. The more we learn the more we see how much of a precision instrument biological life is.

How can phoenix produce the same sequence today that supposedly exists in the human genome unchanged although accumulating over 100,000 generations of mutations? The human genome has many instances of specific sequences and specific locations so we indeed do see that a pandemic will result in specific sequences and specific sites. We would expect shared insertion points and to have a pattern of sequence discrepancies that shadowed the similarity of the genomes as a whole. Partly because of more similar genomes more likely being susceptible to the same virus, and retroviral infection processes will behave more similarly in more similar substrate. I am leaning toward agreement with your statement that insertion point is fairly random, but the process of viral infection yields retrotransposition through site specific integrases and recombinases that result in the specific sequence being directed by orderly mechanisms to a specific location. The insertion gets propogated to every individual because the specific sequence at its ideal location will be found in all of the individuals including there gametes because of the pandemic. These results in a very loud signal in the gene pool. Now how do you get from one individual with an insertion to populations of species with the same integration?
Last edited by NewCreature on Sat Apr 21, 2007 9:52 pm, edited 2 times in total.
NewCreature
Familiar Member
Posts: 32
Joined: Fri Mar 30, 2007 6:32 am

Post by NewCreature »

Well I think I have communicated the course of my hypothesizing about retroviral integration into life. Perhaps one could suggest that the judgments of God at the fall, and perhaps even the changes in life after the destruction by the flood would result in genetic evidences of these changes. It is clear that the genetic matter in the various species has undergone changes since their initial states. Massive outbreaks of retroviral infection can silence genes and alter others. I'm sure that God acted in whatever way he choose, but I can think of no other mechanism whereby this could happen, or how else we would find evidence of it taking place. Actually it is a remarkabley little understood subject.

Retroviral signatures have remained extraordinarily fixed in light of the instability of proviral DNA especially caused by the reverse transcription of the retroviral RNA. If one accepts millions of years it would seem they need to either doubt the time period, or question the observations showing the extraordinary ability of retroviral RNA and proviral DNA to mutate rapidly.

It makes me wonder why a 5,000,000 year old retrovirus hasn't changed remarkably in the time that the entire human species has been able to evolve into its current form. Evolution is so full of holes and complex explanations I can hardly fathom how it can become generally accepted by many people. In any event their remarkable similarity to the sequences in retroviral infections today point to a short period of time.

That should pretty well clarify my position together with my other posts on retroviruses. Now I want to see if I can communicate and be helped to understand the generally accepted common ancestry source of retroviral signatures.

The storybook would say that we are at some point 5 million years ago when large mammals were coming into their prime. Enter the individual. Is this a case of speciation? Apparently this individual must be unique from all other similar individals present on the day the common ancestor became infected. Wouldn't all others in this group be excluded from the genepool?

Clearly I understand very poorly whet is going on at this point, so if anyone has a good grasp of how this process is hypothesized to work I would appreciate a post on it, so I can understand it.

I realize that in order for this to work at all that there must be a minimum of two of these common ancestors. It seems that this would have to be the case for every single case of speciation that has ever taken place. It would seem to me for this to work, that both these individuals would need to have the same proviral signature in the same insertion site so that all their offspring could bear the mark of the HERV. Wouldn't they also need the same mutation that caused them to no longer be able to interbreed with the other gene pool?
NewCreature
Familiar Member
Posts: 32
Joined: Fri Mar 30, 2007 6:32 am

Post by NewCreature »

BGoodForGoodSake wrote:Once a retreoviral material has been injected into the genome it is no longer subject to the accelerated mutation rates of viruses.

Therefore integrated material will mutate at a much slower rate than material within a viral population.

Reverse transcription does not occur once the material has been injected into the host genome!
Retroviral activity is quite poorly understood. The least understood areas of retroviruses is their action in the cell after the cell has become infected and how they operate within the genome. I would say to your first point that the retroviral material has no ability to mutate until it has injected itself into the DNA of its host. I'm not sure how a retroviral population would mutate except by use of the genome of its viable hosts. Reverse transcription can easily happen after a cellular infection. The virus does block the reinsertion of the exact same sequence, but the remarkably error prone process of Reverse transcription yields quite a variety of slightly different retroviral fragments and these slightly different viruses can reinfect the already infected cell. Reverse transcription will again take places within the cell resulting in two infections. This is particularly noted in HIV.

Reverse transcriptase I assure you is working in the genome and on these sequneces even outside of the viral infection. In fact these sequences are often reverse transcribed.

Please see here, and if you would like more information on how reverse transcription is involved you can review the literature on it.

http://en.wikipedia.org/wiki/Reverse_transcriptase
Self-replicating stretches of eukaryotic genomes known as retrotransposons utilise reverse transcriptase to move from one position in the genome to another via a RNA intermediate. They are found abundantly in the genomes of plants and animals. Telomerase is another reverse transcriptase found in many eukaryotes, including humans, which carries its own RNA template; this RNA is used as a template for DNA replication



Also I found this at sciencedirect.com in my travels. IT does indicate that a DNA viral sequences, which is what a provirus would be after reverse transcription, is capable of inserting at a single location.

http://tinyurl.com/2l4utm

Adeno-associated virus (AAV) is a DNA virus that integrates in human cells at a single location on chromosome 19, but curiously, integration of AAV-based vectors results in more widely distributed integration.

This was also in this article

The integration targeting specificities for the HERVs are unknown, so they might have favored initial integration in intergenic regions. However, in this case, it seems likely that selection after integration played an important role. The minority of HERV sequences within genes are oriented opposite to the direction of host gene transcription fully 80% of the time. The reverse orientation of HERVs means that the element-encoded signals for RNA processing (splicing, cleavage, and polyadenylation) do not disrupt expression of the host gene, rendering the inserted HERV sequences relatively benign. Thus, integrated HERVs have apparently been selected at the cellular level after integration to minimize genetic damage to the host genome.
User avatar
BGoodForGoodSake
Ultimate Member
Posts: 2127
Joined: Mon Aug 29, 2005 9:44 am
Christian: No
Location: Washington D.C.

Post by BGoodForGoodSake »

I know you don't fully understand the subject and I think you yourself know you don't, so instead of making assertions and reassertions, please just ask me questions. I would love to answer your questions, and We can even go step by step in your reasoning, so I can help you think more scientifically. I think it would be a wonderful excersize useful to all those reading this thread. :)

Anyways to keep the confusion levels low I will address one subject at a time. To help answer questions relating to and of Phoenix I submit the following.
NewCreature wrote:It makes me wonder why a 5,000,000 year old retrovirus hasn't changed remarkably in the time that the entire human species has been able to evolve into its current form. Evolution is so full of holes and complex explanations I can hardly fathom how it can become generally accepted by many people. In any event their remarkable similarity to the sequences in retroviral infections today point to a short period of time.
Phoenix is a result of HERV-K. Now there are numerous HERV-K insertions in the human genome. All of them have base pair replacements(mutations).

You seem to be making the assumption that in 5 million years that the viral material should be obliterated. But based on what? Observed mutation rates do not support this.
Mutation rates of eukaryotic DNA.

Since this section of the genome is not under the pressures of natural selection it will undergo more mutations than the rest of the genome as a whole, however in 5 million years we do not expect the code to be obliterated.

Now you may wonder if the code is so damaged how did they manage to rebuild the virus? Let me know if you don't understand.

There are multiple copies of HERV-K. As you know a virus will insert its material many times not just once. All of these copies have gathered separate mutations over time. By examining each one and finding the most common base at each position we can reconstruct the virus.

For example lets say we have 5 identical(position wise) sections of HERV-K code as follows.
ATTGGGCA
ACTCGGGA
CACCCGCA
GCACGGCT
AGTTGACC

Since A showed up 3 of the 5 times in the first position we can safely say that the first position most likely was originally an A.

We then choose the most commonly found base at each position resulting in the following.

ACTCGGCA

This is how the Phoenix particle was reconstructed. And to test if the reconstruction is a valid one it was then used to reinfect mammalian cells. The result is that the signature of the infection is identical to the signature found in human beings, including preferential insertion (methylation) site. :!: Note this does not mean identical insertion points.

Several things to note. As you can see the result showed that this particular ERV must have a viral origin. It is highly, highly, highly, :) unlikely that thousands of separate sections of non-coding human DNA will accidently encode for a working virus.

Also another interesting thing to note is that this experiment actually tested evolution. Evolutionary theory states that each individual copy of HERV-K have its own set of mutations. The hypothesis that the reconstruction could even be done rests on this principle. Although this does not prove evolution, it is difficult otherwise to explain, why there are a thousand insertions each of which are modified versions of an ancient virus.
Of course each copy is not identical as some only contain partial insertions. But for simplicity sake I did not go into such details.

Another interesting thing we can do. We are able to date when the insertion must have entered the human genome because each copy of HERV-K has its own mutations. And mutation rates are known. By comparing each section of code to the phoenix we can see how many base pair substitutions were made.

Another thing you brought up regarding HERV-K
NewCreature wrote:
BGoodForGoodSake wrote: The phoenix insertion exhibited the exact same signatures in hamster, cat and human cells. This would seem to be counter to your assertion that the same viral class would exhibit different insertional signatures because of the uniqueness of each species genome.
I am unfamiliar with this observation; do you have any more information on this?. I would suggest that isolated cells in a petri dish do not behave in the same ways as an organism.
Yes this was in the same article you linked to. It's fine to make suggestions, but it is not supported completely by observations. We cannot continually submit hypotheses to support other hypotheses. that is not how science works. We make one hypothesis and try to test it. We try to prove it wrong. What you are doing in essence is changing the subject.
But let us address this "idea". Phoenix is unlikely to even reach target cells today. As it is 5 million years behind in the arms race. So in this sense you are correct. However integration is likely to occur similarly in all these cells because in essence there is virtually no difference between the three animals cells. In other words the only difference is the sequence of the DNA itself. And DNA consist of 4 base pairs only, so it is all repetitive and there is nothing so distinct to even expect individual viral behaviour in cells of different species. Let me know if you didn't understand this. This is why I found that suggestion so surprising.

Now lets revisit your statement.
NewCreature wrote:It makes me wonder why a 5,000,000 year old retrovirus hasn't changed remarkably in the time that the entire human species has been able to evolve into its current form. Evolution is so full of holes and complex explanations I can hardly fathom how it can become generally accepted by many people. In any event their remarkable similarity to the sequences in retroviral infections today point to a short period of time.
As you can see the viral material has undergone mutations. However the redundant copies allowed scientists to run a sort of error correction process.

Also the last sentence mentions a remarkable similarity to signatures to infections today.
There are no infections today but those from the reconstructed Phoenix. The signatures are expected to match!
:)
That shows that the reconstruction was valid, among other things.
It is not length of life, but depth of life. -- Ralph Waldo Emerson
NewCreature
Familiar Member
Posts: 32
Joined: Fri Mar 30, 2007 6:32 am

Post by NewCreature »

BGoodForGoodSake wrote:I know you don't fully understand the subject and I think you yourself know you don't, so instead of making assertions and reassertions, please just ask me questions.
Your post teeters on the edge of silly. Do you suggest I come to you with questions because I don't fully understand the subject so that I can find answers from you who is the one that has the full understanding? You know your post seems to be in part aimed at dismissing what I have said by trying to minimize the authority of the speaker. I'm not sure why you felt the need to sink to that tactic, but you have.


BGoodForGoodSake wrote:Phoenix is a result of HERV-K. Now there are numerous HERV-K insertions in the human genome. All of them have base pair replacements(mutations).

All of these sequences are not the result of separate infections. Retroelements have the ability to propagate throughout the genome, and it is likely that many of the HERV-K sequences are translocations of viral elements. This is a highly error prone process and most probably is the reason for base pair replacements
BGoodForGoodSake wrote:You seem to be making the assumption that in 5 million years that the viral material should be obliterated. But based on what? Observed mutation rates do not support this.
Mutation rates of eukaryotic DNA.

And yet the DNA of the common ancestor was able to pick up enough mutations to become human, and other members of that gene pool were able to mutate into chimps, and gorillas, etc. Why is mutation able to cause numerous speciation events while displaying the inability to mutate a provirus DNA sequence that should be more mutable than encoding genes are?
BGoodForGoodSake wrote: Since this section of the genome is not under the pressures of natural selection it will undergo more mutations than the rest of the genome as a whole, however in 5 million years we do not expect the code to be obliterated.

I never made that assertion. Why are you setting up strawmen?
BGoodForGoodSake wrote: Now you may wonder if the code is so damaged how did they manage to rebuild the virus? Let me know if you don't understand.
Do you think that anyone that doesn't agree with you has trouble understanding? I understand the process, and that is why I asked the questions of you. Because various pieces of HERV-K have translocated throughout the genome we have numerous copies for comparison. It is also quite likely that there are even HERV-K elements from more than one viral outbreak in the human genome. Using all this available information scientists were able to replicate the most likely retrovirus that yielded what we view as HERV-K. The result as we know is the phoenix retrovirus. The signature left by inserting this virus, after removing all the mutations, into living matter is remarkable similar to HERV-K sequences that are suggested to have undergone 5,000,000 years of mutations. This asks the question why is the sequence still the same in a species that has mutated into a different species? Proviral sequences are supposed to be some of the most mutable, yet this suggests the sequence has remained nearly unchanged.
BGoodForGoodSake wrote:Several things to note. As you can see the result showed that this particular ERV must have a viral origin. It is highly, highly, highly, :) unlikely that thousands of separate sections of non-coding human DNA will accidently encode for a working virus.
I don't find myself in agreement here. I would suggest that it indicates the possibility of a retroviral origin of these ERV sequences. When scientists are taking DNA sequences and comparing them and reconstructing a virus from bits of our genome I wouldn't exactly call it an accident. It is clear that they were able to construct a virus from information in our genome on purpose and by design that yielded the same sequence, so I would say it seems likely that a virus is the cause.

BGoodForGoodSake wrote:Also another interesting thing to note is that this experiment actually tested evolution. Evolutionary theory states that each individual copy of HERV-K have its own set of mutations. The hypothesis that the reconstruction could even be done rests on this principle. Although this does not prove evolution, it is difficult otherwise to explain, why there are a thousand insertions each of which are modified versions of an ancient virus.
Of course each copy is not identical as some only contain partial insertions. But for simplicity sake I did not go into such details..
What? Are you serious? This is so full of assumptions I'm not sure how you could suggest that it is experimental evidence for speciation. IT is far more likely that the known process of reverse transcription that has been shown to yield any number of errors, and the ability of these elements to translocate throughout the genome is the causes of what we see.


BGoodForGoodSake wrote: Another interesting thing we can do. We are able to date when the insertion must have entered the human genome because each copy of HERV-K has its own mutations. And mutation rates are known. By comparing each section of code to the phoenix we can see how many base pair substitutions were made.
Please clearly demonstrate this. Also please show how the differnces in base pairs are the result of time and not the result of errors in reverse transcription.
BGoodForGoodSake wrote: Also the last sentence mentions a remarkable similarity to signatures to infections today.
There are no infections today but those from the reconstructed Phoenix. The signatures are expected to match!
:)
That shows that the reconstruction was valid, among other things.
I think I had trouble communicating. I meant that the signature caused by the integration of the phoenix proviral DNA insertion would yield the same sequence as the HERV-K signatures that we see today that have reportedly accumulated 5,000,000 years of mutation. OH and have a great day, I see that you are coming to more and more of an understanding of retroviruses.
User avatar
BGoodForGoodSake
Ultimate Member
Posts: 2127
Joined: Mon Aug 29, 2005 9:44 am
Christian: No
Location: Washington D.C.

Post by BGoodForGoodSake »

NewCreature wrote:Your post teeters on the edge of silly. Do you suggest I come to you with questions because I don't fully understand the subject so that I can find answers from you who is the one that has the full understanding?
No, I didn't mean to come across this way please excuse me.
:(
It was just a reaction to some of the things you posted which completely ignored my posts. I will be a little more patient and will address all your points.
NewCreature wrote:You know your post seems to be in part aimed at dismissing what I have said by trying to minimize the authority of the speaker. I'm not sure why you felt the need to sink to that tactic, but you have.
No I am not trying any tactics, I am not even trying to argue, but to explore the subject.
NewCreature wrote:
BGoodForGoodSake wrote:Phoenix is a result of HERV-K. Now there are numerous HERV-K insertions in the human genome. All of them have base pair replacements(mutations).
All of these sequences are not the result of separate infections. Retroelements have the ability to propagate throughout the genome, and it is likely that many of the HERV-K sequences are translocations of viral elements.
Perhaps I underestimated your level of understanding. However keep in mind that many insertions are latent and do not result in retrotransposons.
NewCreature wrote:This is a highly error prone process and most probably is the reason for base pair replacements.
I would have to disagree, not in the fact that it is error prone, but the idea that it happened recently. If it did then how can all humans share the so many insertion points? And sequences.
NewCreature wrote:And yet the DNA of the common ancestor was able to pick up enough mutations to become human, and other members of that gene pool were able to mutate into chimps, and gorillas, etc. Why is mutation able to cause numerous speciation events while displaying the inability to mutate a provirus DNA sequence that should be more mutable than encoding genes are?
Let's stick to retrovirus and not expand the discussion to proviruses in general. I'm not sure I understand your point here, there are not wholesale differences in those genomes. And many of the differences lies in the material under discussion.
NewCreature wrote:
BGoodForGoodSake wrote: Since this section of the genome is not under the pressures of natural selection it will undergo more mutations than the rest of the genome as a whole, however in 5 million years we do not expect the code to be obliterated.
I never made that assertion. Why are you setting up straw men to discredit me?
I'm not trying to set up strawmen, I may have misunderstood you. What is it you are trying to say? I understood it to mean that after 5 million years we should not be able to reconstruct phoenix. Am I incorrect, please clarify and we'll continue from there.
Also NewCreature I have been sending you Private messages so that we can reduce this sort of misunderstaning in the public thread, I think it would be better to try to clear up points that way therefore the posts don't have to seem so confusing and long.
:) Just a suggestion.
NewCreature wrote:
BGoodForGoodSake wrote: Now you may wonder if the code is so damaged how did they manage to rebuild the virus? Let me know if you don't understand.
Do you think that anyone that doesn't agree with you has trouble understanding?
No. I am sorry if it appears that way. I just want to make sure that I am as clear as I possibly can be. Additionally I was replying to your comment that you did not understand.
NewCreature wrote:You know if you don't have anything to add without these kind of comments perhaps you should move along. I understand the process, and that is why I asked the questions of you. Because various pieces of HERV-K have translocated throughout the genome we have numerous copies for comparison. It is also quite likely that there are even HERV-K elements from more than one viral outbreak in the human genome. Using all this available information scientists were able to replicate the most likely retrovirus that yielded what we view as HERV-K. The result as we know is the phoenix retrovirus. The signature left by inserting this virus, after removing all the mutations, into living matter is remarkable similar to HERV-K sequences that are suggested to have undergone 5,000,000 years of mutations. This asks the question why is the sequence still the same in a species that has mutated into a different species? Proviral sequences are supposed to be some of the most mutable, yet this suggests the sequence has remained nearly unchanged.
I dont follow your reasoning.

The insertions were not mutation free.
The mutations were removed by comparing the multiple copies of ERV's resulting in a reconstruction.
The virus was then allowed to reinfect a host cell.
The signature is not an exact sequence, it is a pattern of integration.
The sequence of the phoenix is not the same as insertions, otherwise a reconstruction would not have been necessary in the first place.
Let me know if you disagree.
NewCreature wrote:
BGoodForGoodSake wrote:Several things to note. As you can see the result showed that this particular ERV must have a viral origin. It is highly, highly, highly, :) unlikely that thousands of separate sections of non-coding human DNA will accidently encode for a working virus.
I don't find myself in agreement here. I would suggest that it indicates the possibility of a retroviral origin of these ERV sequences. When scientists are taking DNA sequences and comparing them and reconstructing a virus from bits of our genome I wouldn't exactly call it an accident. It is clear that they were able to construct a virus from information in our genome on purpose and by design that yielded the same sequence, so I would say it seems likely that a virus is the cause.
I'm sorry I seem to have miread your post. It seems here you are saying you disagree that HERV-K must be viral in origin. But then state that HERV-K is viral in origin. Please let me know if I misread this.
NewCreature wrote:
BGoodForGoodSake wrote:Also another interesting thing to note is that this experiment actually tested evolution. Evolutionary theory states that each individual copy of HERV-K have its own set of mutations. The hypothesis that the reconstruction could even be done rests on this principle. Although this does not prove evolution, it is difficult otherwise to explain, why there are a thousand insertions each of which are modified versions of an ancient virus.
Of course each copy is not identical as some only contain partial insertions. But for simplicity sake I did not go into such details..
What? Are you serious? This is so full of assumptions I'm not sure how you could suggest that it is experimental evidence for speciation.
No, I did not say speciation, I said evolution, and more specifically the idea that mututations accumulate over time.
NewCreature wrote:IT is far more likely that the known process of reverse transcription that has been shown to yield any number of errors, and the ability of these elements to translocate throughout the genome is the causes of what we see.
This may be so. But mutations surely were continued to be added since the time of the insertions.
NewCreature wrote:
BGoodForGoodSake wrote: Another interesting thing we can do. We are able to date when the insertion must have entered the human genome because each copy of HERV-K has its own mutations. And mutation rates are known. By comparing each section of code to the phoenix we can see how many base pair substitutions were made.
Please clearly demonstrate this. Also please show how the differnces in base pairs are the result of time and not the result of errors in reverse transcription.
1. Calculate the average number of substitutions which result from reverse transcription.
2. Calculate the percentage of change in DNA minus the first value.
3. Calculate the average rate of mutations.
4. Use the value from number 3 to estimate the dates of insertion.
But you have a point here, #1 will make this method less reliable.

We can continue and reconstruct phoenix from and old world monkey lets say. Do the same calculation, and see if they agree.
NewCreature wrote:I think I had trouble communicating. I meant that the signature caused by the integration of the phoenix proviral DNA insertion would yield the same sequence as the HERV-K signatures that we see today that have reportedly accumulated 5,000,000 years of mutation. OH and have a great day, I see that you are coming to more and more of an understanding of retroviruses.
Thank You.
:)
The signatures match after the mutations are taken into account.
I still don't think I understand what you are trying to say here.
The insertions do not yield the same sequence, how could they as the sequence of phoenix had to be reconstructed?

Think of it like a stamp. In this example a little girl has a toy stamp of the Bratz. Lets say we have a smudged imprint with several specks of crayon over top. You come along and like the design but you're not sure which of her stamps created this imprint. So you go through her collection and make neat nice imprints with each. Finally you analyze each one and find that the Bratz imprints signature matches the smudged imprint made by her.

By signature, here we are talking about qualities which help define this imprint.

In the same way the viral signature is used to help identify all of the various insertions which grouped under HERV-K.

And finally this viral signature is used to verify the reconstruction.
NewCreature
Familiar Member
Posts: 32
Joined: Fri Mar 30, 2007 6:32 am

Post by NewCreature »

I just wanted to give a quick response to your most recent post. First I'm also sorry for letting our contention slip away from the contentious points and allowing the contention to become more personal.

The bulk of your post was in reference to the Phoenix virus, so I just have a few comments to help clarify my thinking on that Virus. Fist I would like to comment that nearly half of the matter that makes us who we are, that is our DNA, is comprised of reverse transcripted RNA. The role of this half of our genome is quite poorly understood, in fact it is little over 25 years since the first retroviral sequences was mapped. This half of the DNA does appear to have some remarkably stable pieces of information given the fluidity of many of the process involved in insertions, deletions, mutations, transcriptions, etc etc. IT has been noticed that RNA molecules are sometimes present at synapses, and there really is quite a lot of theory about this half of the human. I am sure that our understanding of the subject is quite imperfect. Sadly this results in hypothesizing with a paltry source of observations to strengthen the ideas. Before I go to far afield, I will address a few comments on the Phoenix virus.

From my reading on the Phoenix virus my understanding is that when it was used to infect cells it left a genetic sequence that is the same as one of the HERV-K sequences. The same sequence was not located in that cells DNA in the same spot as we find that sequence in the human genome.

I do disagree that HERV-K MUST be viral in origin, but I don't know that it is not viral in origin. I am not completely convinced that the HERVs are viral in origin; but of course they may very well be viral in origin. IT should be noted that we don't find naturally occurring counterparts to most of the ERVs. I don't have any problem suggesting or hypothesizing about the viral origins of HERVs, but I do resist the idea that they MUST be.

The main point that we seem at odds about with respect to Phoenix is the signature that phoenix leaves in the infected cells and how they compare to existing HERV-K sequences.

HERV-K retroelements were comparison analyzed and an RNA strand was devised from the most popular sequences in HERV-K retroelements.

This RNA strand was packaged in retroviral machinery and delivered into the cell. Once this happened the RNA strand was reverse transcripted into a proviral DNA strand and integrated into the host cells DNA with the same sequence as an existing HERV-K sequence.

It is not understood how the new sequence would propagate through the genome of a living human.
User avatar
BGoodForGoodSake
Ultimate Member
Posts: 2127
Joined: Mon Aug 29, 2005 9:44 am
Christian: No
Location: Washington D.C.

Post by BGoodForGoodSake »

NewCreature wrote:The bulk of your post was in reference to the Phoenix virus, so I just have a few comments to help clarify my thinking on that Virus. Fist I would like to comment that nearly half of the matter that makes us who we are, that is our DNA, is comprised of reverse transcripted RNA. The role of this half of our genome is quite poorly understood, ...I am sure that our understanding of the subject is quite imperfect. Sadly this results in hypothesizing with a paltry source of observations to strengthen the ideas. Before I go to far afield, I will address a few comments on the Phoenix virus.
I disagree, you are overgeneralizing our ignorance. We should discuss what we do know, and acknowledge what information we lack specifically.
NewCreature wrote:From my reading on the Phoenix virus my understanding is that when it was used to infect cells it left a genetic sequence that is the same as one of the HERV-K sequences.
Again this is incorrect, the signature was the same, the sequence cannot be the same because of the mutations present in the original insertions.

"The resulting provirus sequence contains, as expected, ORFs for all of the HERV-K(HML2)-encoded proteins (Gag, Pro, Pol, Env, and the accessory Rec protein), with gag, pro, and pol separated by —1 frameshifts. Noteworthily, this consensus provirus is distinct from each of the sequences used to generate it, with at least 20 amino acid changes on the overall sequences" (1)
NewCreature wrote:The same sequence was not located in that cells DNA in the same spot as we find that sequence in the human genome.
Yes, I hope this clarifies insertion affinity in general.
NewCreature wrote:I do disagree that HERV-K MUST be viral in origin, but I don't know that it is not viral in origin. I am not completely convinced that the HERVs are viral in origin; but of course they may very well be viral in origin. IT should be noted that we don't find naturally occurring counterparts to most of the ERVs. I don't have any problem suggesting or hypothesizing about the viral origins of HERVs, but I do resist the idea that they MUST be.

The main point that we seem at odds about with respect to Phoenix is the signature that phoenix leaves in the infected cells and how they compare to existing HERV-K sequences.
The signature is as follows. If I keep this up I may end up posting the entire article.
:P
"In all cases, we found complete LTRs with short target-site duplications (TSDs) bordering the newly inserted proviruses (Fig. 5), thus confirming that the infection process is canonical. Rather surprisingly, the TSDs were of a variable length (one of 5 bp and two of 6 bp), which was unexpected since well-characterized retroviral integrases yield TSDs of a fixed length (for review, see Brown 1997). However, our data are consistent with the structures observed for the HERV-K(HML2) endogenous copies of the human genome, since, even if the most common length of the endogenous HERV-K proviruses TSD is 6 bp, we could identify, both in the literature (Barbulescu et al. 1999; Turner et al. 2001) and databases (our unpublished observation), a few elements (three of 15 full-length proviruses) bordered by perfect TSD of only 5 bp (see the endogenous K115 provirus in the lower part of Fig. 5 for an illustration). Finally, we mapped the three Phoenix insertion sites using the UCSC Genome Browser and found that integration occurred within (for two of three) or close to (<20 kb, for one of three) genes. Although the number of target sites that we characterized is low, and thus cannot reveal subtle insertion site preferences, the identified loci are consistent with the preferences observed for classical retroviruses that favor insertion in transcribed regions (for review, see Bushman 2003)."(1)
Signature is not exact sequence, but rather the pattern of integration.
NewCreature wrote:HERV-K retroelements were comparison analyzed and an RNA strand was devised from the most popular sequences in HERV-K retroelements.

This RNA strand was packaged in retroviral machinery and delivered into the cell. Once this happened the RNA strand was reverse transcripted into a proviral DNA strand and integrated into the host cells DNA with the same sequence as an existing HERV-K sequence.
Not to nitpick here but we must be as exact as possible. Not the exact same sequences, again the viral material was reconstructed. I wanted to keep this simple for the general readers but I think for the purposes of our discussion we need to involve every detail.
NewCreature wrote:It is not understood how the new sequence would propagate through the genome of a living human.
What is meant by this, we have a fairly good understanding of how the viral material finds its way into the genome. It's included in this papers thesis.
"We also show that this element amplifies via an extracellular pathway involving reinfection, at variance with the non-LTR-retrotransposons (LINEs, SINEs) or LTR-retrotransposons, thus recapitulating ex vivo the molecular events responsible for its dissemination in the host genomes."(1) Intrestingly this has a bearing on the signature discussion above. The viral signature is in fact the result of insertion, propogation, and reinfection, along with transposition of retrotransposal elements.

One question, how can HERV-K's viral origins still be in doubt?

1. Dewannieux, M, Harper, F, Richaud, A, Letzelter, C, Ribet, D, Pierron, G & Heidmann, T. (2006) Genome Res 16, 1548—1556.
It is not length of life, but depth of life. -- Ralph Waldo Emerson
NewCreature
Familiar Member
Posts: 32
Joined: Fri Mar 30, 2007 6:32 am

Post by NewCreature »

I'm not sure how to communicate what we don't know other than by generalizations. If I could be specific about the things we don't know I would have knowledge of what we don't know. There really is poor understanding of processes involved after the initial retroviral infection. We do understand pretty well how retroviral infection happens, and how the provirus is formed. The copy and paste functions that allow retroelements with or without LTRs to propagate throughout the human genome are understood at the highest level, but there is overall a poor understanding of the nature of the process, and the plethora of mechanisms that influence retrotransposition.
BGoodForGoodSake wrote: Again this is incorrect, the signature was the same, the sequence cannot be the same because of the mutations present in the original insertions.
OK I'm good with that Phoenix integrates in a petri dish with the exact signature of existing HERV-K elements. IT is not clear how the sequences would change in a living human. Many changes to the sequence would result from transposition and interaction with specific substrates, transposases, recombinases, and other cellular mechanisms. Many of the mutations visible in existing HERV-K sequnces could happen during reverse transcription to the RNA intermediary during retrotranslocation.
BGoodForGoodSake wrote:
NewCreature wrote:The same sequence was not located in that cells DNA in the same spot as we find that sequence in the human genome.
Yes, I hope this clarifies insertion affinity in general.
It would be unwise to generalize from one instance in the laboratory. Again it has been shown that a proviral element can insert in a specific place on a specific chromosome. Clearly the mechanism exists. Remember that shortly after integration a retrovirus becomes a provirus. There are site specific recombinases and integrases as well.
BGoodForGoodSake wrote:What is meant by this, we have a fairly good understanding of how the viral material finds its way into the genome.
I agree but that is where the understanding gets cloudy. The propagation once inserted is a bit more fuzzy. This is a young field compared to many other areas of science, but we are gaining knowledge rapidly. IF we are patient perhaps many of our contentious points will be more clearly demonstrated in the coming years.

You asked how I can question the Viral Origin of HERV-K. It very well may be viral in origin. Phoenix simply took information that naturally exists in the human genome and packaged it in a retrovirus and inserted it through that mechanism. Nearly half of the human functioning genome is reverse transcribed. IT is becoming more and more clear that these are important elements of our life. Further I would point out again that most retroelements don't have naturally occurring exogenous retroviruses.

I would ask a question in response. What observations demonstrate that HERV-K must be from a virus?
Post Reply